LSAT and Law School Admissions Forum

Get expert LSAT preparation and law school admissions advice from PowerScore Test Preparation.

User avatar
 Dave Killoran
PowerScore Staff
  • PowerScore Staff
  • Posts: 5853
  • Joined: Mar 25, 2011
|
#84955
Complete Question Explanation
(The complete setup for this game can be found here: lsat/viewtopic.php?t=6605)

The correct answer choice is (D)

As discussed during the analysis of the first rule, T must always run in one of the four races. Thus, answer choice (D) is correct.
 mollyquillin
  • Posts: 15
  • Joined: Jun 03, 2020
|
#78145
Why must the coach select T to run in this race? I'm confused why the answer is not A (which is that Q must be selected). Could someone please explain?
 Paul Marsh
PowerScore Staff
  • PowerScore Staff
  • Posts: 290
  • Joined: Oct 15, 2019
|
#78348
Hi Molly!

So we know from our Setup that 4 of the 5 runners are selected. That means 4 of the people will run, and only 1 will sit out.

Our first Rule tells us that if Q is selected to run, then T must also be selected and run in the very next race. So whenever Q is selected, T must be selected too! We can write this as a conditional if that helps:
Q :arrow: T
And the contrapositive of that conditional would be:
NOT T :arrow: NOT Q

So if T sits out, then that means Q has to sit out too. But remember what I said up above: we know from our Setup that 4 people have to run, and only 1 can sit out. So it's not possible for T and Q to both sit out! That means T always has to be selected.

Hope that helps!

Get the most out of your LSAT Prep Plus subscription.

Analyze and track your performance with our Testing and Analytics Package.